The proof of Time Scaling, Laplace transform

Click For Summary
The discussion focuses on understanding the transition from the left-hand side to the right-hand side of equation 6.96 in the context of time scaling and the Laplace transform. The key steps involve manipulating the exponential term by introducing a constant factor 'a' and changing the variable of integration. By substituting and simplifying, the integral is transformed, allowing for the application of the Laplace transform properties. The clarification provided significantly aids in grasping the underlying mathematical concepts. This exchange highlights the importance of detailed step-by-step explanations in complex mathematical discussions.
killahammad
Messages
5
Reaction score
0
Hi

I understand most of the steps in the determination of the time scale. But i don't really understand the step in equation 6.96.

The first attachment is the full details of the time scale, and the second attachment is the part which I am stuck on.

I just want to know, how they get from the LHS to the RHS of the equation

Thanks for any help :)
 

Attachments

  • scaling.png
    scaling.png
    12.3 KB · Views: 3,279
  • scaling2.png
    scaling2.png
    2 KB · Views: 1,454
Last edited:
Physics news on Phys.org
to go from
\int_0^\infty e^{-st}f(at)dt
You do 2 things: Multiply -st in the exponential by a/a= 1 and rewrite (st)(a/a)= (s/a)(at); also write (a/a)dt= (1/a)(adt). Since a is a constant, we can take the 1/a outside the integral and write adt= d(at). That gives you
\frac{1}{a}\int_0^\infty e^{-\frac{s}{a}}(at)f(at)d(at)
Now, put in a new variable: let t'= at. Then dt'= d(at). When t= 0, u= t' and, as t goes to infinity, t' goes to infinity. In the variable t', f(at)= f(t') so we have
\frac{1}{a}\int_0^\infty e^{(s/a)t'} f(t)dt'
 
thank you very much, that helped me alot
 
Question: A clock's minute hand has length 4 and its hour hand has length 3. What is the distance between the tips at the moment when it is increasing most rapidly?(Putnam Exam Question) Answer: Making assumption that both the hands moves at constant angular velocities, the answer is ## \sqrt{7} .## But don't you think this assumption is somewhat doubtful and wrong?

Similar threads

  • · Replies 1 ·
Replies
1
Views
1K
  • · Replies 4 ·
Replies
4
Views
3K
  • · Replies 4 ·
Replies
4
Views
3K
  • · Replies 8 ·
Replies
8
Views
2K
Replies
9
Views
2K
  • · Replies 2 ·
Replies
2
Views
1K
  • · Replies 14 ·
Replies
14
Views
2K
  • · Replies 19 ·
Replies
19
Views
3K
  • · Replies 2 ·
Replies
2
Views
3K
Replies
7
Views
2K